Wyznacz wartość oczekiwaną i wariancje

Definicja klasyczna. Prawdopodobieństwo warunkowe i całkowite. Zmienne losowe i ich parametry. Niezależność. Prawa wielkich liczb oraz centralne twierdzenia graniczne i ich zastosowania.
maniek980
Użytkownik
Użytkownik
Posty: 10
Rejestracja: 16 gru 2017, o 16:23
Płeć: Mężczyzna
Lokalizacja: Kielce
Podziękował: 2 razy

Wyznacz wartość oczekiwaną i wariancje

Post autor: maniek980 »

Cześć. Pierwszy raz coś piszę korzystając z LaTeX-a, bądźcie wyrozumiali.
ZADANIE
Dystrybuanta zmiennej losowej \(\displaystyle{ \mathrm{X}}\) ma postać \(\displaystyle{ F(x)=1-e^{-\lambda*\pi*x^{2}}}\), gdzie \(\displaystyle{ \lambda >0, x>0}\).
Wyznacz \(\displaystyle{ \mathbb{E}}\)\(\displaystyle{ \mathrm{X}}\), \(\displaystyle{ \mathbb{E}}\)\(\displaystyle{ \mathrm{X^{2}}}\) oraz Var\(\displaystyle{ \mathrm{VarX}}\) (wariancje).
I teraz tak, policzyłem pochodną z dystrybuanty żeby mieć funkcje gęstości
\(\displaystyle{ f(x)=2x \pi \lambda e^{-\lambda \pi x^{2}}}\)
No i chce skorzystać z wzorów EX=\(\displaystyle{ \int_{- \infty }^{+ \infty }x*f(x)}\) itd.
Ktoś pomoże z tą całką
\(\displaystyle{ \int_{- \infty }^{+ \infty }2x^{2} \pi \lambda e^{-\lambda \pi x^{2}}dx}\)
Albo podpowie co jest źle w moim rozumowaniu?
Awatar użytkownika
Janusz Tracz
Użytkownik
Użytkownik
Posty: 4065
Rejestracja: 13 sie 2016, o 15:01
Płeć: Mężczyzna
Lokalizacja: hrubielowo
Podziękował: 80 razy
Pomógł: 1392 razy

Re: Wyznacz wartość oczekiwaną i wariancje

Post autor: Janusz Tracz »

Tą całkę daje się policzyć "jawnie".

\(\displaystyle{ \mathbb{E}\left[ \mathrm{X}\right] =2\pi \lambda\int_{- \infty }^{+ \infty }x^{2}e^{-\lambda \pi x^{2}} \mbox{d}x=-\int_{- \infty }^{+ \infty }x \mbox{d}\left( e^{-\lambda \pi x^{2}}\right)}\)

I dalej przez części.
Ostatnio zmieniony 11 lis 2018, o 13:20 przez Janusz Tracz, łącznie zmieniany 1 raz.
Awatar użytkownika
Premislav
Użytkownik
Użytkownik
Posty: 15687
Rejestracja: 17 sie 2012, o 13:12
Płeć: Mężczyzna
Lokalizacja: Warszawa
Podziękował: 196 razy
Pomógł: 5220 razy

Re: Wyznacz wartość oczekiwaną i wariancje

Post autor: Premislav »

Na razie wygląda OK. Przez części:
\(\displaystyle{ \int_{}^{} 2x^2\pi \lambda e^{-\lambda \pi x^2}\,\dd x=\\= \int_{}^{} x\left(-e^{-\lambda \pi x^2} \right)'\,\dd x=\\=x\cdot \left( -e^{-\lambda \pi x^2}\right) + \int_{}^{} e^{-\lambda \pi x^2}\,\dd x}\)
i wstawiając granice całkowania \(\displaystyle{ \pm \infty}\), mamy z uwagi na
\(\displaystyle{ \lim_{x \to \infty} -xe^{-\lambda\pi x^2}= \lim_{x \to -\infty} -xe^{-\lambda\pi x^2}=0}\):
\(\displaystyle{ \int_{-\infty}^{+\infty} 2x^2\pi \lambda e^{-\lambda \pi x^2}\,\dd x=\\= \int_{-\infty}^{+\infty} e^{-\lambda \pi x^2}\,\dd x}\)
Teraz w tej ostatniej całce podstaw \(\displaystyle{ \sqrt{2\pi \lambda}x=t}\), a otrzymasz:
\(\displaystyle{ \int_{-\infty}^{+\infty} \frac{1}{\sqrt{2\pi \lambda}}e^{-\frac{t^2}{2}}\,\dd t=\\=\frac{\sqrt{2\pi}}{\sqrt{2\pi \lambda}}=\frac{1}{\sqrt{\lambda}}}\)
Skorzystałem z własności rozkładu normalnego (właściwie można to było zrobić od razu, a nie sprowadzać do \(\displaystyle{ \mathcal{N}(0,1),}\) ale wtedy pewnie bym się walnął w obliczeniach); jeśli nie miałeś rozkładu normalnego, to trzeba by wykazać, że
\(\displaystyle{ \int_{-\infty}^{+\infty} e^{-\frac{t^2}{2}}\,\dd t=\sqrt{2\pi}}\):
jest to całkiem wykonalne (standardowy trick to obliczenie kwadratu tego przez tw. Fubiniego), ale na razie to pominę, bo może miałeś rozkład normalny.
Natomiast w odróżnieniu od tego
\(\displaystyle{ \mathbf{E}(X^2)}\) właśnie wychodzi całkiem zgrabnie przez części i nie trzeba nawet znać rozkładu normalnego.
maniek980
Użytkownik
Użytkownik
Posty: 10
Rejestracja: 16 gru 2017, o 16:23
Płeć: Mężczyzna
Lokalizacja: Kielce
Podziękował: 2 razy

Wyznacz wartość oczekiwaną i wariancje

Post autor: maniek980 »

Dzięki za wyjaśnienie, to z obliczeniem \(\displaystyle{ \mathbb{E}}\)\(\displaystyle{ \mathrm{X^{2}}}\) oraz \(\displaystyle{ \mathrm{VarX}}\) już sobie poradzę.
Miałem rozkład normalny, a o tym twierdzeniu poczytam w internecie
@edit
Jednak mam problem z \(\displaystyle{ \mathbb{E}}\)\(\displaystyle{ \mathrm{X^{2}}}\)
\(\displaystyle{ \mathrm{EX^{2}}=\int_{-\infty}^{+\infty}2\pi \lambda x^{3} e^{-\pi \lambda x^{2}}dx=\begin{vmatrix}
f=x^{2}\hspace{1cm}f'=2x\\ g'=2\pi \lambda xe^{-\pi \lambda x^{2}} \hspace{1cm} g=-e^{-\pi \lambda x^{2}}
\end{vmatrix}=\n -x^{2}e^{-\pi \lambda x^{2}}+\int_{-\infty}^{+\infty}2xe^{-\pi \lambda x^{2}}dx}\)

i zostaje
\(\displaystyle{ \int_{-\infty}^{+\infty}2\pi \lambda x^{3} e^{-\pi \lambda x^{2}}dx=\int_{-\infty}^{+\infty}2xe^{-\pi \lambda x^{2}}dx= \lim_{ a \to-\infty } -\frac{1}{\lambda \pi e^{\pi \lambda x^{2}}}\line(0,1){1,5em}_{a}^{0}+\lim_{ b \to+\infty } -\frac{1}{\lambda \pi e^{\pi \lambda x^{2}}}\line(0,1){1,5em}_{0}^{b}=- \frac{1}{\pi \lambda}+ \frac{1}{\pi \lambda}=0}\)
Wychodzi mi ujemna wariancja. Miałem sporą przerwe od całek pewnie coś źle liczę, widzi ktoś błąd?
Awatar użytkownika
Premislav
Użytkownik
Użytkownik
Posty: 15687
Rejestracja: 17 sie 2012, o 13:12
Płeć: Mężczyzna
Lokalizacja: Warszawa
Podziękował: 196 razy
Pomógł: 5220 razy

Re: Wyznacz wartość oczekiwaną i wariancje

Post autor: Premislav »

Ja to jestem taką pałą, że szkoda słów. Przecież gęstość musi być nieujemna, a ja sobie beztrosko całkuję od \(\displaystyle{ -\infty}\) do \(\displaystyle{ +\infty}\)
Sorry, przecież nawet to w treści zadania zaznaczono, że \(\displaystyle{ x>0}\), powinno być tak:
\(\displaystyle{ \mathbf{E}(X)= \int_{{\red 0}}^{+\infty} 2x^2\pi e^{-\lambda \pi x^2}\,\dd x}\)
Liczy się to dalej właściwie prawie tak samo, jak pokazałem (całkowanie przez części), jak nikt mnie nie uprzedzi, to za jakiś kwadrans wstawię obliczenia.
maniek980
Użytkownik
Użytkownik
Posty: 10
Rejestracja: 16 gru 2017, o 16:23
Płeć: Mężczyzna
Lokalizacja: Kielce
Podziękował: 2 razy

Wyznacz wartość oczekiwaną i wariancje

Post autor: maniek980 »

Skopiowałem Twój skrypt, bo słabo mi idzie LaTeX
\(\displaystyle{ \int_{}^{} 2x^2\pi \lambda e^{-\lambda \pi x^2}\,\dd x=\\= \int_{}^{} x\left(-e^{-\lambda \pi x^2} \right)'\,\dd x=\\=x\cdot \left( -e^{-\lambda \pi x^2}\right) + \int_{}^{} e^{-\lambda \pi x^2}\,\dd x}\)
i wstawiając granice całkowania \(\displaystyle{ 0, \infty}\), mamy z uwagi na (dla zera wprost, a dla nieskończoności pójdzie de l'Hospitala
\(\displaystyle{ \lim_{x \to 0} -xe^{-\lambda\pi x^2}= \lim_{x \to \infty} -xe^{-\lambda\pi x^2}=0}\):
\(\displaystyle{ \int_{0}^{+\infty} 2x^2\pi \lambda e^{-\lambda \pi x^2}\,\dd x=\\= \int_{0}^{+\infty} e^{-\lambda \pi x^2}\,\dd x}\)
Stosuję podstawienie \(\displaystyle{ \sqrt{2\pi \lambda}x=t}\) i teraz:
\(\displaystyle{ \int_{0}^{+\infty} \frac{1}{\sqrt{2\pi \lambda}}e^{-\frac{t^2}{2}}}\) Mógłbyś tu rozpisać co dalej sie dzieje?


\(\displaystyle{ \mathrm{EX^{2}}= \int_{0}^{+\infty} 2x^3\pi \lambda e^{-\lambda \pi x^2}== \int_{0}^{+\infty} x^{2}\left(-e^{-\lambda \pi x^2} \right)'\,\dd x=\\=x^{2}\cdot \left( -e^{-\lambda \pi x^2}\right) + \int_{0}^{+\infty} 2xe^{-\lambda \pi x^2}\,\dd x=}\)
Tutaj znowu ten sam myk z \(\displaystyle{ 0,\infty}\)
\(\displaystyle{ \int_{0}^{+\infty} 2x^3\pi \lambda e^{-\lambda \pi x^2}\,\dd x= \int_{0}^{+\infty}2x e^{-\lambda \pi x^2}\,\dd x=}\)(przez podstawienie wyjdzie całka)\(\displaystyle{ =\lim_{ b \to+\infty } -\frac{1}{\lambda \pi e^{\pi \lambda x^{2}}}\line(0,1){2em}_{0}^{b}= \frac{1}{\pi \lambda}}\)
Awatar użytkownika
Premislav
Użytkownik
Użytkownik
Posty: 15687
Rejestracja: 17 sie 2012, o 13:12
Płeć: Mężczyzna
Lokalizacja: Warszawa
Podziękował: 196 razy
Pomógł: 5220 razy

Re: Wyznacz wartość oczekiwaną i wariancje

Post autor: Premislav »

Sorry, plany się trochę zmieniły…
Tak, znając rozkład normalny, wiemy, że
\(\displaystyle{ \int_{-\infty}^{+\infty} \frac{1}{\sqrt{2\pi}}e^{-\frac{t^2}{2}}\,\dd t=1}\),
poza tym funkcja \(\displaystyle{ f(t)= \frac{1}{\sqrt{2\pi}}e^{-\frac{t^2}{2}}}\) jest parzysta, zatem
\(\displaystyle{ \int_{-\infty}^{0} \frac{1}{\sqrt{2\pi}}e^{-\frac{t^2}{2}}\,\dd t=\int_{0}^{+\infty} \frac{1}{\sqrt{2\pi}}e^{-\frac{t^2}{2}}\,\dd t}\),
czyli
\(\displaystyle{ \int_{-\infty}^{+\infty} \frac{1}{\sqrt{2\pi}}e^{-\frac{t^2}{2}}\,\dd t=\frac 1 2}\)
i
\(\displaystyle{ \int_{-\infty}^{+\infty} \frac{1}{\sqrt{2\pi\lambda }}e^{-\frac{t^2}{2}}\,\dd t=\frac 1 {2\sqrt{\lambda}}}\)

Natomiast \(\displaystyle{ \mathbf{E}(X^2)}\) obliczyłeś poprawnie.
ODPOWIEDZ